Each group of order 8 has a subgroup of order 2 and a subgroup of order 4.












6












$begingroup$


So, I was trying to prove the following theorem:




Let $G$ be a group of order $8$. So $G$ has a subgroup of order $2$ and a subgroup of order $4$.




First I proved that if a group has a finite even order, it has an element $g_0$ of order $2$. So $H={g_0,e}$ is a group of order $2$. Now I'm trying to find the group of order $4$. By Lagrange theorem I can understand that:



$$|G|=|H|cdot |G,:,H| Rightarrow |G,:,H|=4$$



So we have $4$ Cosets. But how to continue from here? I feel like I miss the last one-two lines of the proof.










share|cite|improve this question











$endgroup$








  • 1




    $begingroup$
    Do you know that nilpotent groups (and in particular $p$-groups) have non-trivial centres? If yes, it would be better and more instructive to prove that nilpotent groups (and in particular $p$-groups) have subgroups of each possible order. (In fact, this holds for a broader class of groups.)
    $endgroup$
    – the_fox
    Feb 15 at 17:37
















6












$begingroup$


So, I was trying to prove the following theorem:




Let $G$ be a group of order $8$. So $G$ has a subgroup of order $2$ and a subgroup of order $4$.




First I proved that if a group has a finite even order, it has an element $g_0$ of order $2$. So $H={g_0,e}$ is a group of order $2$. Now I'm trying to find the group of order $4$. By Lagrange theorem I can understand that:



$$|G|=|H|cdot |G,:,H| Rightarrow |G,:,H|=4$$



So we have $4$ Cosets. But how to continue from here? I feel like I miss the last one-two lines of the proof.










share|cite|improve this question











$endgroup$








  • 1




    $begingroup$
    Do you know that nilpotent groups (and in particular $p$-groups) have non-trivial centres? If yes, it would be better and more instructive to prove that nilpotent groups (and in particular $p$-groups) have subgroups of each possible order. (In fact, this holds for a broader class of groups.)
    $endgroup$
    – the_fox
    Feb 15 at 17:37














6












6








6





$begingroup$


So, I was trying to prove the following theorem:




Let $G$ be a group of order $8$. So $G$ has a subgroup of order $2$ and a subgroup of order $4$.




First I proved that if a group has a finite even order, it has an element $g_0$ of order $2$. So $H={g_0,e}$ is a group of order $2$. Now I'm trying to find the group of order $4$. By Lagrange theorem I can understand that:



$$|G|=|H|cdot |G,:,H| Rightarrow |G,:,H|=4$$



So we have $4$ Cosets. But how to continue from here? I feel like I miss the last one-two lines of the proof.










share|cite|improve this question











$endgroup$




So, I was trying to prove the following theorem:




Let $G$ be a group of order $8$. So $G$ has a subgroup of order $2$ and a subgroup of order $4$.




First I proved that if a group has a finite even order, it has an element $g_0$ of order $2$. So $H={g_0,e}$ is a group of order $2$. Now I'm trying to find the group of order $4$. By Lagrange theorem I can understand that:



$$|G|=|H|cdot |G,:,H| Rightarrow |G,:,H|=4$$



So we have $4$ Cosets. But how to continue from here? I feel like I miss the last one-two lines of the proof.







abstract-algebra group-theory






share|cite|improve this question















share|cite|improve this question













share|cite|improve this question




share|cite|improve this question








edited Feb 15 at 17:58









Shaun

9,268113684




9,268113684










asked Feb 15 at 17:23









BadukBaduk

493




493








  • 1




    $begingroup$
    Do you know that nilpotent groups (and in particular $p$-groups) have non-trivial centres? If yes, it would be better and more instructive to prove that nilpotent groups (and in particular $p$-groups) have subgroups of each possible order. (In fact, this holds for a broader class of groups.)
    $endgroup$
    – the_fox
    Feb 15 at 17:37














  • 1




    $begingroup$
    Do you know that nilpotent groups (and in particular $p$-groups) have non-trivial centres? If yes, it would be better and more instructive to prove that nilpotent groups (and in particular $p$-groups) have subgroups of each possible order. (In fact, this holds for a broader class of groups.)
    $endgroup$
    – the_fox
    Feb 15 at 17:37








1




1




$begingroup$
Do you know that nilpotent groups (and in particular $p$-groups) have non-trivial centres? If yes, it would be better and more instructive to prove that nilpotent groups (and in particular $p$-groups) have subgroups of each possible order. (In fact, this holds for a broader class of groups.)
$endgroup$
– the_fox
Feb 15 at 17:37




$begingroup$
Do you know that nilpotent groups (and in particular $p$-groups) have non-trivial centres? If yes, it would be better and more instructive to prove that nilpotent groups (and in particular $p$-groups) have subgroups of each possible order. (In fact, this holds for a broader class of groups.)
$endgroup$
– the_fox
Feb 15 at 17:37










3 Answers
3






active

oldest

votes


















8












$begingroup$

Well, you can show that $G$ has a subgroup of order 4 directly. If $G$ has an element of order 4 or 8 then we are already done [why?]



Otherwise let $pi$ and $alpha$ be two distinct elements in $G$ of order 2. So $pi = pi^{-1}$ and $alpha = alpha^{-1}$. Then $pialpha$ must have order 2 as well, lest there be an element of order 4 or 8 which would imply that we are done. This implies that $pi alpha = alpha pi$. [Indeed, for any elements $a,b,c$ in any group $G'$, if $ab=ac$ then $b$ must equal $c$. But here $(pi alpha)(pi alpha) = 1$, while $(pi alpha)(alpha pi) = (pi alpha)(alpha^{-1}pi^{-1}) = 1$. So indeed, $pi alpha = alpha pi$.] This implies that $H doteq {1,alpha, pi, pialpha }$ is closed under composition, and as every element in $H$ has its inverse in $H$, it follows that $H$ is a subgroup, and has order 4.






share|cite|improve this answer











$endgroup$





















    6












    $begingroup$

    If $G$ has no subgroup of order $4$, then every $gne e$ has order $2$. Then for any $g,h$,
    $$
    e=(gh)^2=ghgh=ghg^{-1}h^{-1},
    $$
    i.e. $G$ is abelian. Then ${e,g,h,gh}$ would be a subgroup of order $4$.






    share|cite|improve this answer









    $endgroup$













    • $begingroup$
      why every $gneq e$ has order $2$ if $G$ does not have subgroup of order $4$? Also why $ghgh=ghg^{-1}h^{-1}$?
      $endgroup$
      – Baduk
      Feb 15 at 18:21












    • $begingroup$
      @Baduk: Apply Lagrange's Theorem. Also, we have $x^{-1}=x$ iff $x^2=e$. (Why?)
      $endgroup$
      – Shaun
      Feb 15 at 18:50





















    3












    $begingroup$

    Hint: No group has exactly two elements of order two.




    Assume no subgroup of order four exists. Apply Lagrange's Theorem. Apply the hint. See @Mike's answer.




    Reference for the lemma that is the hint:



    Exercise 4.61 of Gallian's "Contemporary Abstract Algebra (Eighth Edition)"






    share|cite|improve this answer











    $endgroup$













    • $begingroup$
      While this link may answer the question, it is better to include the essential parts of the answer here and provide the link for reference. Link-only answers can become invalid if the linked page changes. - From Review
      $endgroup$
      – Gibbs
      Feb 15 at 17:58






    • 1




      $begingroup$
      @Gibbs, I've included a reference.
      $endgroup$
      – Shaun
      Feb 15 at 18:01











    Your Answer





    StackExchange.ifUsing("editor", function () {
    return StackExchange.using("mathjaxEditing", function () {
    StackExchange.MarkdownEditor.creationCallbacks.add(function (editor, postfix) {
    StackExchange.mathjaxEditing.prepareWmdForMathJax(editor, postfix, [["$", "$"], ["\\(","\\)"]]);
    });
    });
    }, "mathjax-editing");

    StackExchange.ready(function() {
    var channelOptions = {
    tags: "".split(" "),
    id: "69"
    };
    initTagRenderer("".split(" "), "".split(" "), channelOptions);

    StackExchange.using("externalEditor", function() {
    // Have to fire editor after snippets, if snippets enabled
    if (StackExchange.settings.snippets.snippetsEnabled) {
    StackExchange.using("snippets", function() {
    createEditor();
    });
    }
    else {
    createEditor();
    }
    });

    function createEditor() {
    StackExchange.prepareEditor({
    heartbeatType: 'answer',
    autoActivateHeartbeat: false,
    convertImagesToLinks: true,
    noModals: true,
    showLowRepImageUploadWarning: true,
    reputationToPostImages: 10,
    bindNavPrevention: true,
    postfix: "",
    imageUploader: {
    brandingHtml: "Powered by u003ca class="icon-imgur-white" href="https://imgur.com/"u003eu003c/au003e",
    contentPolicyHtml: "User contributions licensed under u003ca href="https://creativecommons.org/licenses/by-sa/3.0/"u003ecc by-sa 3.0 with attribution requiredu003c/au003e u003ca href="https://stackoverflow.com/legal/content-policy"u003e(content policy)u003c/au003e",
    allowUrls: true
    },
    noCode: true, onDemand: true,
    discardSelector: ".discard-answer"
    ,immediatelyShowMarkdownHelp:true
    });


    }
    });














    draft saved

    draft discarded


















    StackExchange.ready(
    function () {
    StackExchange.openid.initPostLogin('.new-post-login', 'https%3a%2f%2fmath.stackexchange.com%2fquestions%2f3114176%2feach-group-of-order-8-has-a-subgroup-of-order-2-and-a-subgroup-of-order-4%23new-answer', 'question_page');
    }
    );

    Post as a guest















    Required, but never shown

























    3 Answers
    3






    active

    oldest

    votes








    3 Answers
    3






    active

    oldest

    votes









    active

    oldest

    votes






    active

    oldest

    votes









    8












    $begingroup$

    Well, you can show that $G$ has a subgroup of order 4 directly. If $G$ has an element of order 4 or 8 then we are already done [why?]



    Otherwise let $pi$ and $alpha$ be two distinct elements in $G$ of order 2. So $pi = pi^{-1}$ and $alpha = alpha^{-1}$. Then $pialpha$ must have order 2 as well, lest there be an element of order 4 or 8 which would imply that we are done. This implies that $pi alpha = alpha pi$. [Indeed, for any elements $a,b,c$ in any group $G'$, if $ab=ac$ then $b$ must equal $c$. But here $(pi alpha)(pi alpha) = 1$, while $(pi alpha)(alpha pi) = (pi alpha)(alpha^{-1}pi^{-1}) = 1$. So indeed, $pi alpha = alpha pi$.] This implies that $H doteq {1,alpha, pi, pialpha }$ is closed under composition, and as every element in $H$ has its inverse in $H$, it follows that $H$ is a subgroup, and has order 4.






    share|cite|improve this answer











    $endgroup$


















      8












      $begingroup$

      Well, you can show that $G$ has a subgroup of order 4 directly. If $G$ has an element of order 4 or 8 then we are already done [why?]



      Otherwise let $pi$ and $alpha$ be two distinct elements in $G$ of order 2. So $pi = pi^{-1}$ and $alpha = alpha^{-1}$. Then $pialpha$ must have order 2 as well, lest there be an element of order 4 or 8 which would imply that we are done. This implies that $pi alpha = alpha pi$. [Indeed, for any elements $a,b,c$ in any group $G'$, if $ab=ac$ then $b$ must equal $c$. But here $(pi alpha)(pi alpha) = 1$, while $(pi alpha)(alpha pi) = (pi alpha)(alpha^{-1}pi^{-1}) = 1$. So indeed, $pi alpha = alpha pi$.] This implies that $H doteq {1,alpha, pi, pialpha }$ is closed under composition, and as every element in $H$ has its inverse in $H$, it follows that $H$ is a subgroup, and has order 4.






      share|cite|improve this answer











      $endgroup$
















        8












        8








        8





        $begingroup$

        Well, you can show that $G$ has a subgroup of order 4 directly. If $G$ has an element of order 4 or 8 then we are already done [why?]



        Otherwise let $pi$ and $alpha$ be two distinct elements in $G$ of order 2. So $pi = pi^{-1}$ and $alpha = alpha^{-1}$. Then $pialpha$ must have order 2 as well, lest there be an element of order 4 or 8 which would imply that we are done. This implies that $pi alpha = alpha pi$. [Indeed, for any elements $a,b,c$ in any group $G'$, if $ab=ac$ then $b$ must equal $c$. But here $(pi alpha)(pi alpha) = 1$, while $(pi alpha)(alpha pi) = (pi alpha)(alpha^{-1}pi^{-1}) = 1$. So indeed, $pi alpha = alpha pi$.] This implies that $H doteq {1,alpha, pi, pialpha }$ is closed under composition, and as every element in $H$ has its inverse in $H$, it follows that $H$ is a subgroup, and has order 4.






        share|cite|improve this answer











        $endgroup$



        Well, you can show that $G$ has a subgroup of order 4 directly. If $G$ has an element of order 4 or 8 then we are already done [why?]



        Otherwise let $pi$ and $alpha$ be two distinct elements in $G$ of order 2. So $pi = pi^{-1}$ and $alpha = alpha^{-1}$. Then $pialpha$ must have order 2 as well, lest there be an element of order 4 or 8 which would imply that we are done. This implies that $pi alpha = alpha pi$. [Indeed, for any elements $a,b,c$ in any group $G'$, if $ab=ac$ then $b$ must equal $c$. But here $(pi alpha)(pi alpha) = 1$, while $(pi alpha)(alpha pi) = (pi alpha)(alpha^{-1}pi^{-1}) = 1$. So indeed, $pi alpha = alpha pi$.] This implies that $H doteq {1,alpha, pi, pialpha }$ is closed under composition, and as every element in $H$ has its inverse in $H$, it follows that $H$ is a subgroup, and has order 4.







        share|cite|improve this answer














        share|cite|improve this answer



        share|cite|improve this answer








        edited Feb 15 at 17:43

























        answered Feb 15 at 17:35









        MikeMike

        4,171412




        4,171412























            6












            $begingroup$

            If $G$ has no subgroup of order $4$, then every $gne e$ has order $2$. Then for any $g,h$,
            $$
            e=(gh)^2=ghgh=ghg^{-1}h^{-1},
            $$
            i.e. $G$ is abelian. Then ${e,g,h,gh}$ would be a subgroup of order $4$.






            share|cite|improve this answer









            $endgroup$













            • $begingroup$
              why every $gneq e$ has order $2$ if $G$ does not have subgroup of order $4$? Also why $ghgh=ghg^{-1}h^{-1}$?
              $endgroup$
              – Baduk
              Feb 15 at 18:21












            • $begingroup$
              @Baduk: Apply Lagrange's Theorem. Also, we have $x^{-1}=x$ iff $x^2=e$. (Why?)
              $endgroup$
              – Shaun
              Feb 15 at 18:50


















            6












            $begingroup$

            If $G$ has no subgroup of order $4$, then every $gne e$ has order $2$. Then for any $g,h$,
            $$
            e=(gh)^2=ghgh=ghg^{-1}h^{-1},
            $$
            i.e. $G$ is abelian. Then ${e,g,h,gh}$ would be a subgroup of order $4$.






            share|cite|improve this answer









            $endgroup$













            • $begingroup$
              why every $gneq e$ has order $2$ if $G$ does not have subgroup of order $4$? Also why $ghgh=ghg^{-1}h^{-1}$?
              $endgroup$
              – Baduk
              Feb 15 at 18:21












            • $begingroup$
              @Baduk: Apply Lagrange's Theorem. Also, we have $x^{-1}=x$ iff $x^2=e$. (Why?)
              $endgroup$
              – Shaun
              Feb 15 at 18:50
















            6












            6








            6





            $begingroup$

            If $G$ has no subgroup of order $4$, then every $gne e$ has order $2$. Then for any $g,h$,
            $$
            e=(gh)^2=ghgh=ghg^{-1}h^{-1},
            $$
            i.e. $G$ is abelian. Then ${e,g,h,gh}$ would be a subgroup of order $4$.






            share|cite|improve this answer









            $endgroup$



            If $G$ has no subgroup of order $4$, then every $gne e$ has order $2$. Then for any $g,h$,
            $$
            e=(gh)^2=ghgh=ghg^{-1}h^{-1},
            $$
            i.e. $G$ is abelian. Then ${e,g,h,gh}$ would be a subgroup of order $4$.







            share|cite|improve this answer












            share|cite|improve this answer



            share|cite|improve this answer










            answered Feb 15 at 17:33









            SongSong

            14.4k1635




            14.4k1635












            • $begingroup$
              why every $gneq e$ has order $2$ if $G$ does not have subgroup of order $4$? Also why $ghgh=ghg^{-1}h^{-1}$?
              $endgroup$
              – Baduk
              Feb 15 at 18:21












            • $begingroup$
              @Baduk: Apply Lagrange's Theorem. Also, we have $x^{-1}=x$ iff $x^2=e$. (Why?)
              $endgroup$
              – Shaun
              Feb 15 at 18:50




















            • $begingroup$
              why every $gneq e$ has order $2$ if $G$ does not have subgroup of order $4$? Also why $ghgh=ghg^{-1}h^{-1}$?
              $endgroup$
              – Baduk
              Feb 15 at 18:21












            • $begingroup$
              @Baduk: Apply Lagrange's Theorem. Also, we have $x^{-1}=x$ iff $x^2=e$. (Why?)
              $endgroup$
              – Shaun
              Feb 15 at 18:50


















            $begingroup$
            why every $gneq e$ has order $2$ if $G$ does not have subgroup of order $4$? Also why $ghgh=ghg^{-1}h^{-1}$?
            $endgroup$
            – Baduk
            Feb 15 at 18:21






            $begingroup$
            why every $gneq e$ has order $2$ if $G$ does not have subgroup of order $4$? Also why $ghgh=ghg^{-1}h^{-1}$?
            $endgroup$
            – Baduk
            Feb 15 at 18:21














            $begingroup$
            @Baduk: Apply Lagrange's Theorem. Also, we have $x^{-1}=x$ iff $x^2=e$. (Why?)
            $endgroup$
            – Shaun
            Feb 15 at 18:50






            $begingroup$
            @Baduk: Apply Lagrange's Theorem. Also, we have $x^{-1}=x$ iff $x^2=e$. (Why?)
            $endgroup$
            – Shaun
            Feb 15 at 18:50













            3












            $begingroup$

            Hint: No group has exactly two elements of order two.




            Assume no subgroup of order four exists. Apply Lagrange's Theorem. Apply the hint. See @Mike's answer.




            Reference for the lemma that is the hint:



            Exercise 4.61 of Gallian's "Contemporary Abstract Algebra (Eighth Edition)"






            share|cite|improve this answer











            $endgroup$













            • $begingroup$
              While this link may answer the question, it is better to include the essential parts of the answer here and provide the link for reference. Link-only answers can become invalid if the linked page changes. - From Review
              $endgroup$
              – Gibbs
              Feb 15 at 17:58






            • 1




              $begingroup$
              @Gibbs, I've included a reference.
              $endgroup$
              – Shaun
              Feb 15 at 18:01
















            3












            $begingroup$

            Hint: No group has exactly two elements of order two.




            Assume no subgroup of order four exists. Apply Lagrange's Theorem. Apply the hint. See @Mike's answer.




            Reference for the lemma that is the hint:



            Exercise 4.61 of Gallian's "Contemporary Abstract Algebra (Eighth Edition)"






            share|cite|improve this answer











            $endgroup$













            • $begingroup$
              While this link may answer the question, it is better to include the essential parts of the answer here and provide the link for reference. Link-only answers can become invalid if the linked page changes. - From Review
              $endgroup$
              – Gibbs
              Feb 15 at 17:58






            • 1




              $begingroup$
              @Gibbs, I've included a reference.
              $endgroup$
              – Shaun
              Feb 15 at 18:01














            3












            3








            3





            $begingroup$

            Hint: No group has exactly two elements of order two.




            Assume no subgroup of order four exists. Apply Lagrange's Theorem. Apply the hint. See @Mike's answer.




            Reference for the lemma that is the hint:



            Exercise 4.61 of Gallian's "Contemporary Abstract Algebra (Eighth Edition)"






            share|cite|improve this answer











            $endgroup$



            Hint: No group has exactly two elements of order two.




            Assume no subgroup of order four exists. Apply Lagrange's Theorem. Apply the hint. See @Mike's answer.




            Reference for the lemma that is the hint:



            Exercise 4.61 of Gallian's "Contemporary Abstract Algebra (Eighth Edition)"







            share|cite|improve this answer














            share|cite|improve this answer



            share|cite|improve this answer








            edited Feb 15 at 18:09

























            answered Feb 15 at 17:31









            ShaunShaun

            9,268113684




            9,268113684












            • $begingroup$
              While this link may answer the question, it is better to include the essential parts of the answer here and provide the link for reference. Link-only answers can become invalid if the linked page changes. - From Review
              $endgroup$
              – Gibbs
              Feb 15 at 17:58






            • 1




              $begingroup$
              @Gibbs, I've included a reference.
              $endgroup$
              – Shaun
              Feb 15 at 18:01


















            • $begingroup$
              While this link may answer the question, it is better to include the essential parts of the answer here and provide the link for reference. Link-only answers can become invalid if the linked page changes. - From Review
              $endgroup$
              – Gibbs
              Feb 15 at 17:58






            • 1




              $begingroup$
              @Gibbs, I've included a reference.
              $endgroup$
              – Shaun
              Feb 15 at 18:01
















            $begingroup$
            While this link may answer the question, it is better to include the essential parts of the answer here and provide the link for reference. Link-only answers can become invalid if the linked page changes. - From Review
            $endgroup$
            – Gibbs
            Feb 15 at 17:58




            $begingroup$
            While this link may answer the question, it is better to include the essential parts of the answer here and provide the link for reference. Link-only answers can become invalid if the linked page changes. - From Review
            $endgroup$
            – Gibbs
            Feb 15 at 17:58




            1




            1




            $begingroup$
            @Gibbs, I've included a reference.
            $endgroup$
            – Shaun
            Feb 15 at 18:01




            $begingroup$
            @Gibbs, I've included a reference.
            $endgroup$
            – Shaun
            Feb 15 at 18:01


















            draft saved

            draft discarded




















































            Thanks for contributing an answer to Mathematics Stack Exchange!


            • Please be sure to answer the question. Provide details and share your research!

            But avoid



            • Asking for help, clarification, or responding to other answers.

            • Making statements based on opinion; back them up with references or personal experience.


            Use MathJax to format equations. MathJax reference.


            To learn more, see our tips on writing great answers.




            draft saved


            draft discarded














            StackExchange.ready(
            function () {
            StackExchange.openid.initPostLogin('.new-post-login', 'https%3a%2f%2fmath.stackexchange.com%2fquestions%2f3114176%2feach-group-of-order-8-has-a-subgroup-of-order-2-and-a-subgroup-of-order-4%23new-answer', 'question_page');
            }
            );

            Post as a guest















            Required, but never shown





















































            Required, but never shown














            Required, but never shown












            Required, but never shown







            Required, but never shown

































            Required, but never shown














            Required, but never shown












            Required, but never shown







            Required, but never shown







            Popular posts from this blog

            Biblatex bibliography style without URLs when DOI exists (in Overleaf with Zotero bibliography)

            ComboBox Display Member on multiple fields

            Is it possible to collect Nectar points via Trainline?